Teilchen in einer 1-D-Box und das Korrespondenzprinzip

Betrachten Sie das Teilchen in einer 1-d-Box, wir kennen die Lösungen davon sehr gut. Ich würde gerne sehen, wie das Korrespondenzprinzip in diesem Fall funktioniert, wenn wir die Positionswahrscheinlichkeitsdichtefunktion (pdf) des Teilchens betrachten.

Im klassischen Fall, bei fehlendem Potenzial, ist es fair genug, die Position pdf einzunehmen P c l a s s ich c des Teilchens innerhalb der Box konstant und außerhalb davon null sein.

Im Quantenfall wissen wir, dass es so ist

P q u a n t u m ( x ) = 2 L Sünde 2 ( n π x L )
innerhalb der Box und null außerhalb.

Ein Bild sagt mehr als tausend Worte, also schnappe ich mir hier ein Bild von einer Universitätswebsite (eigentlich Google), für das ich den Autoren dankbar bin.

Teilchen in einer 1-D-Box

Aus dem Bild können wir das sehen, wenn wir eine ausreichend große Quantenzahl betrachten n , wir sehen, dass die P Q u a n t u m wackelt immer noch und egal wie groß n wir denken, es wackelt ständig und macht sich nicht wirklich die Mühe, sich zu konvergieren P c l a s s ich c , aber der Trost ist, dass sie im durchschnittlichen Sinne übereinstimmen.

Idealerweise würde ich bei einem so grundlegenden Problem wie diesem die Funktion erwarten P q u a n t u m ( x ) zu konvergieren P c l a s s ich c ( x ) , punktuell . Verlange ich zu viel? Es gibt unendlich viele Arten von P q u a n t u m passend P c l a s s ich c im durchschnittlichen Sinne, was bedeuten würde, dass es unendlich viele quantenmechanische Theorien geben kann, die dem Korrespondenzprinzip im durchschnittlichen Matching gehorchen, was meiner Meinung nach keine sehr ansprechende Eigenschaft einer Theorie ist.

Meine Frage ist, was können wir tun, damit das PDF punktuell mit dem klassischen PDF konvergiert?

Wunderbare Frage. Ich denke darüber nach, dass die einzige Einschränkung, die wir der Quantentheorie auferlegen, darin besteht, dass sie mit dem beobachteten Spektrum verschiedener physikalischer Größen übereinstimmen und das klassische Ergebnis reproduzieren sollte, bei dem die Plank-Konstante auf Null geht. Aber ob es punktuell passen sollte, weiß ich nicht. Warten auf eine gute Erklärung.

Antworten (2)

Es ist erwähnenswert, dass das Quanten-PDF zwar immer noch schnelle Schwankungen aufweist, diese jedoch sehr hochfrequent werden, und wenn Sie die Verteilung mit einem physikalischen Instrument messen , führen Sie eine Auflösung ein.

Irgendwann wird diese Auflösung viel größer als die Wellenlänge der Fluxationen und Sie erkennen tatsächlich die klassische Verteilung.

Sie verlassen sich also auf fehlende Messgeräte! Was wäre, wenn sie punktuell zusammenlaufen würden, würde dann das Argument des fehlenden Messgerätes aus der QM-Szene verschwinden?

Meine Frage ist, was können wir tun, damit das PDF punktuell mit dem klassischen PDF konvergiert

Antwort : Nichts, wenn Sie die Quantenprinzipien akzeptieren.

Unten sehen Sie ein weiteres Beispiel mit dem harmonischen Oszillator, mit N = 50 , und Sie sehen dasselbe Verhalten, über das Sie sich Sorgen machen (klassisches Verhalten in Blau, Quantenverhalten in Rot):

Der grundlegende Grund ist, dass es in der Quantenmechanik um Korrelationsamplituden und Wahrscheinlichkeitsamplituden geht, sodass klassische statistische Konzepte wie Korrelationswahrscheinlichkeiten oder Wahrscheinlichkeiten eine "Baby" -Ansicht der physikalischen Welt sind. Ein „Wackeln“ ist also nicht zu vermeiden. Der einfachste Ansatz ist, dass Wahrscheinlichkeitsamplituden oder Korrelationsamplituden für gebundene Zustände „natürlich“ oszillieren, was in Wahrscheinlichkeiten als Oszillationen zwischen Null- und „Maximal“-Werten umgeschrieben wurde, während für große Quantenzahlen ein „Mittelwert“ aussieht eine klassische Wahrscheinlichkeit.

Geben Sie hier die Bildbeschreibung ein

Frohe Weihnachten! und danke für die Antwort! ... Was ich an Ihrer Antwort interessant finde, ist der Satz "Wahrscheinlichkeiten sind eine "Baby" -Ansicht der physikalischen Welt. Sie können also das "Wackeln" nicht vermeiden ... Nun Meine große Frage ist ein bisschen hypothetisch. Angenommen, zum Beispiel, wenn das QM mit all seinen Erklärungen der physikalischen Welt in Takt ist (Spektren des Wasserstoffatoms, harmonischer Oszillator, bla ... bla ... bla ...) ... ABER es ist pdf konvergierte punktuell mit dem von pdf der klassischen Theorie in Limit n geht ins Unendliche, genau wie ich es in dieser Frage erwartet hatte ... Physikalisch und philosophisch, was würde das bedeuten?
Rajesh D: Es würde weder physikalisch noch philosophisch etwas bedeuten, denn Sie würden ein Modell vorschlagen, das nicht mit der beobachtbaren Realität in Einklang zu bringen ist. Die Quantenmechanik konvergiert nur deshalb mit der klassischen, weil die Wellenlängen fein genug werden, um nach klassischen Maßstäben vernachlässigbar zu werden. Spin ist ein weiteres Beispiel: Sie beginnen nur mit Auf und Ab und fügen dann mit zunehmender Spin-Zahl immer feinere Winkel hinzu. Die größte Botschaft von QM ist, dass es im Grunde nur Wellen sind – und Wellen bestehen darauf, Dinge wie Nullpunkte zu haben, die nicht glatt klassisch sein können.
@TerryBollinger: Angenommen, wenn das QM (oder eine neue Version davon) mit all seinen Erklärungen der physischen Welt in Takt ist, aber sein PDF immer noch punktuell mit dem klassischen konvergiert ...
@Trimok: Sie haben die blaue Kurve berechnet, was ist der formale Ausdruck, den Sie verwendet haben, in klassischen Begriffen, meine ich x , v , t etc., v Potenzial sein.
@RajeshD: Tatsächlich müssen Sie eine bestimmte Lösung der Bewegungsgleichung schreiben, also im Fall des harmonischen Oszillators: x = EIN cos ω t . Hier liegt das Potenzial v ( x ) = 1 2 m ω 2 x 2 . Die Gesamtenergie ist E = 1 2 m ω 2 EIN 2 . Die Wahrscheinlichkeit ist also proportional zum Kehrwert der Geschwindigkeit p ( x ) 1 | x ˙ ( x ) | . Also muss man sich ausdrücken x ˙ als Funktion von x , hier hast du | x ˙ | = EIN ω | Sünde ω t | = EIN ω 1 x 2 EIN 2 , so endlich p ( x ) 1 1 x 2 EIN 2 ,...
@RajeshD: ... der Proportionalitätskoeffizient wird definiert durch EIN + EIN p ( x ) d x = 1
@Trimok: Vielen Dank. Die Wahrscheinlichkeit ist an diesem Punkt umgekehrt proportional zum Kehrwert der Geschwindigkeit. Danke dafür und +1.